Deci, în $ {\ bf R} ^ {n \ times p} $ avem produsul interior Frobenius dat de $$ \ langle A, B \ rangle = \ text {tr} (A ^ TB) $$

care poate fi interpretat ca produsul interior euclidian pe $ {\ bf R} ^ {np } $. Înțeleg că toate produsele interioare din $ {\ bf R} ^ {np} $ pot fi scrise ca $$ a ^ TPb $$ pentru $ P $ pozitiv-definit. Cel mai bun lucru pe care l-aș putea face încercând să extind produsul interior Frobenius pe $ {\ bf R} ^ {n \ ori p} $ este ceva de forma $$ \ langle A, B \ rangle = \ sum_ {i = 1} ^ N \ text {tr} ((X_iAY_i) ^ T (X_iBY_i)) $$ pentru $ X_i \ în {\ bf R} ^ {m_i \ times n} $ și $ Y_i \ in {\ bf R} ^ {p \ times q_i} $ toate rangul complet. Cu toate acestea, aș dori să știu dacă acest lucru acoperă toate produsele interioare de pe $ {\ bf R} ^ {np} $ sau dacă poate este mai complex decât este necesar din cauza concedierilor.

Pot găsi matrice corespunzătoare $ P $ pentru orice produs interior specific matricei luând baza standard pentru $ {\ bf R} ^ {n \ ori p} $ și formând matricea

\ begin {bmatrix} \ langle E_1 , E_1 \ rangle & \ langle E_1, E_2 \ rangle & \ dots & \ langle E_1, E_ {np} \ rangle \\ \ langle E_2, E_1 \ rangle & \ langle E_2, E_2 \ rangle & & \ vdots \\ \ vdots & & \ ddots \\ \ langle E_ {np }, E_1 \ rangle & \ dots & \ dots & \ langle E_ {np }, E_ {np} \ rangle \ end {bmatrix}

dar nu știu dacă forma generală pentru un produs interior matricial pe care l-am dat mai sus acoperă toate matricile pozitive-definite $ P $.

Actualizare:

versiunea mai nouă a acestei întrebări pe MathOverflow: https://mathoverflow.net/questions/229675/extending-the-trace-inner-product-to-all-matrix-real-inner-products

Comentarii

  • Bine ați venit la SciComp.SE! Aceasta este o întrebare interesantă, dar pare mult mai potrivită pentru math.stackexchange.com . (Cu excepția cazului în care ' există o conexiune la o problemă științifică de calcul, ' m lipsește, caz în care ' ar fi grozav dacă ați putea adăuga acest lucru.)
  • @ChristianClason, ' este legat de optimizarea pe varietăți matriciale cu valori Riemannian, deoarece Riemannian metricele sunt produse interioare pe spațiul tangent. Este ' aproape sigur prea avansat pentru Math.SE, singurul alt loc adecvat ar fi MathOverflow. Este posibil să fi găsit ceea ce cred că este o soluție pe care o pot posta ca răspuns odată ce fac munca dezordonată de a dovedi că este o soluție, dar dacă ' doriți să migrați asta pentru MathOverflow. I ' m ok cu asta. ' voi adăuga contextul de optimizare atunci când voi avea o șansă.
  • Matricea $ P $ trebuie, de asemenea, să fie simetrică, nu doar pozitivă definită.
  • @WolfgangBangerth, pozitiv-definit se înțelege că implică simetric.
  • Nu pentru toți autorii pozitiv-definit înseamnă implică simetrie.

Răspuns

Puteți vedea un produs interior ca o operație $ f (a, b) = \ left < a, b \ right > $, adică este o funcție biliniară care (i) returnează un număr non-negativ, (ii) satisface relația $ f (a, b) = f (b, a) $.

Pentru vectorii $ a, b \ in \ mathbb R ^ n $, toate funcțiile biliniare care satisfac aceste proprietăți pot fi scrise ca $$ f (a, b) = \ sum_ {i, j = 1 } ^ n a_i P_ {ij} b_j $$ unde $ P $ este simetric și pozitiv definit. Pentru matricile $ a, b \ in \ mathbb R ^ {n \ times p} $, toate aceste funcții pot fi scrise ca $$ f (a, b) = \ sum_ {i, k = 1} ^ n \ sum_ { j, l = 1} ^ p a_ {ij} P_ {ijkl} b_ {kl} $$ unde acum $ P $ este un tensor de rangul 4 care este simetric în sensul că $ P_ {ijkl} = P_ {klij} $ și pozitiv definit în sensul că $ f (a, a) > 0 $ pentru toate $ a \ neq 0 $.

Întrebarea dvs. se reduce la dacă fiecare $ P $ care îndeplinește astfel de condiții poate fi scris un formular care rezultă din vectorii $ X_i, Y_i $. Cred că răspunsul la aceasta este nu. Acest lucru este pur și simplu așa deoarece (pentru simplitate presupunând $ n = p $) $ P $ simetric are (asimptotic) $ n ^ 4/2 $ grade de libertate, în timp ce vectorii $ n $ $ X_i, Y_i $ au doar $ 2n ^ 2 $ grade de libertate. Cu alte cuvinte, nu cred că pentru $ n $ suficient de mare, abordarea dvs. are suficient de multe grade de libertate.

Comentarii

  • I de fapt, cred că răspunsul este da, ' o să redau această întrebare la depășirea matematică cu rezultatele mele actualizate.
  • Da, argumentul tău că numărul de parametri crește trimestrial în spațiul interior al produsului vector, în timp ce doar patru în spațiul interior al matricei este convingător, totuși, deoarece spațiul este în cele din urmă finit, ar trebui să putem depăși acest lucru crescând în mod corespunzător $ N $.
  • Scuzele mele Am postat o versiune mai nouă a acestei întrebări pe MathOverflow, însă ' este suficient de actualizată Am considerat că este adecvat, aici este linkul în cazul în care doriți să vă transferați răspunsul acolo sau să vă actualizați răspunsul pe baza versiunii mai noi. mathoverflow.net/questions/229675/…
  • @Thoth Rețineți că @ ChristianClason a recomandat să vă postați întrebarea pe math.stackexchange.com, nu pe mathoverflow.net. Acestea sunt două site-uri diferite cu scopuri și audiențe diferite.
  • @FedericoPoloni, da, știu, și dacă citești ce am scris, i-am spus că am crezut că este prea avansat pentru Math.SE și că este puțin probabil să obții un răspuns acolo.

Lasă un răspuns

Adresa ta de email nu va fi publicată. Câmpurile obligatorii sunt marcate cu *